Hi. Thanks for the reply. Doing the following suggests that the uncertainty is ~139Ω.
Looking online, I found a formula that corresponds to this answer.
http://physics.stackexchange.com/questions/111507/relative-error-of-equivalent-resistance-of-resistors-in-parallel
I would therefore have to...